What’s the range of the given function?

I marked that answer by accident btw

Whats The Range Of The Given Function?I Marked That Answer By Accident Btw

Answers

Answer 1

The range of the given function is - 2


Related Questions

Simplify:......................................................

Answers

Answer:

...

Step-by-step explanation:...

The cut off number is 2x-1

HELP HELP HELP
Solve this

Answers

Answer:

What is the cos theta for, i would use sin to solve for theta and then we would get 41.25 degrees.

Step-by-step explanation:

N/A questions does not exist

Answers

Answer:

ok

Step-by-step explanation:

why did you put it

Help me with this answer I don’t it

Answers

Answer:

f(-2) = g(-2) this is the answer

Find the value of -6 + 8(-3) ÷ 4
please help me .

Answers

Answer:

-12

Step-by-step explanation:

cause from the rule

we should always solve from bracket so -3×8= -24

and -24/4= -6

and -6-6

= -12

Find the measure of the arc or angle indicated. Assume that lines which appear tangent are tangent.

Answers

Given:

In the given circle, the measure of major arc TV is 220 degrees.

To find:

The measure of angle TUV.

Solution:

In a circle, the measure central angle of arc is equal to its corresponding arc measure and according to central angle theorem, the measure of subtended angle on an arc is half of the measure of central angle of that arc.

So, the measure of subtended angle is half of the measure of its corresponding arc.

[tex]m\angle TUV=\dfrac{1}{2}\times \text{Measure of major arc TV}[/tex]

[tex]m\angle TUV=\dfrac{1}{2}\times 220^\circ[/tex]

[tex]m\angle TUV=110^\circ[/tex]

Therefore, the measure of angle TUV is 110 degrees.


Ibrahim heeft een bijbaantje op de markt. Hij berekent zijn inkomsten met de formule
inkomsten in €=5+3,50 x tijd in uren. Leg de formule uit.

Answers

Answer:

Ibrahim gets 5 fixed and 3.5 per hour.

Step-by-step explanation:

Ibrahim has a side job at the market. He calculates his income with the formula income in € = 5 + 3.50 x,  time in hours. Explain the formula.

Here, the fixed income is 5.

the income per hour is 3.5.

So, Ibrahim gets 5 fixed and 3.5 per hour.

If f(x) = x -2 and g(x) = 2x – 6, then g(4)/f(3) =​

Answers

Answer:

Step-by-step explanation:

(2×4-6)/(3-2)=2

Answer:

[tex]{ \tt{f(x) = x - 2}} \\ { \bf{f(3) = 3 - 2 = 1}} \\ \\ { \tt{g(x) = 2x - 6}} \\ { \bf{g(4) = 2(4) - 6 = 2}} \\ \\ { \boxed{ \tt{ \frac{g(4)}{f(3)} = \frac{2}{1} = 2}}}[/tex]

11x+7y=17
solve for y

Answers

[tex]\implies {\blue {\boxed {\boxed {\purple {\sf {\: y = \frac{17 - 11x}{7} }}}}}}[/tex]

[tex]\large\mathfrak{{\pmb{\underline{\red{Step-by-step\:explanation}}{\red{:}}}}}[/tex]

[tex]\\11x + 7y = 17[/tex]

[tex] \\➺ \: 7y = 17 - 11x[/tex]

[tex]\\➺ \: y = \frac{17 - 11x}{7} [/tex]

[tex]\bold{ \green{ \star{ \orange{Mystique35}}}}⋆[/tex]

HELP ME PLEASEEEEEEEEEEEEEE

Answers

Answer:

a

Step-by-step explanation:

Given points (-3;-6), G(3; -2) and H(6; 1); determine:
(a) The equation of line FG

Answers

Answer:

The equation of line FG is [tex]y = \frac{2}{3}x - 4[/tex]

Step-by-step explanation:

Equation of a line:

The equation of a line has the following format:

[tex]y = mx + b[/tex]

In which m is the slope and b is the y-intercept.

F(-3;-6), G(3; -2)

When we have two points, the slope is given by the change in y divided by the change in x. So

Change in y : -2 - (-6) = -2 + 6 = 4

Change in x: 3 - (-3) = 3 + 3 = 6

Slope: [tex]m = \frac{4}{6} = \frac{2}{3}[/tex]

So

[tex]y = \frac{2}{3}x + b[/tex]

Finding b:

(3; -2) means that when [tex]x = 3, y = -2[/tex]. We use this to find b.

[tex]y = \frac{2}{3}x + b[/tex]

[tex]-2 = \frac{2}{3}(3) + b[/tex]

[tex]2 + b = -2[/tex]

[tex]b = -4[/tex]

The equation of line FG is [tex]y = \frac{2}{3}x - 4[/tex]

Which number's estimate written as a single digit times a power of 10 will have a negative exponent?

Answers

105 i hope this helps if not then i’m sorry

A geneticist conducts an experiment with peas, one sample of offspring consisted of 450 green peas and 157 yellow peas. Based on these results, estimate the probability of getting an offspring pea that is green.

Answers

Answer: 0.738

Step-by-step explanation:

11. Mendelian Genetics. When Mendel conducted his famous genetics experiments with peas, one sample of offspring consisted of 428 green peas and 152 yellow peas. Based on those results, estimate the probability of getting an offspring pea that is green. Is the result reasonably close to the value that was expected?

p0 = 428/(428 + 152) = 0.737931

If you round your answer of 0.737931 to three decimals you will

get 0.738.

Which rules of exponents will be used to evallate this expression? Select three options.

Answers

I need the answer choices and the expression you’re referring to

Six consecutive numbers add up to a total of 69.what is the highest of these numbers?

Answers

Answer:

14

Step-by-step explanation:

9+10+11+12+13+14=69

The [tex]HIGHEST[/tex] of these numbers is [tex]14[/tex]

Let the [tex]CONSECUTIVE[/tex] numbers be : [tex]a, a+1, a +2,a+3,a+4,a+5[/tex]

Taking the sum :

[tex]a + a + 1 + a + 2 + a + 3 + a + 4 + a + 5 = 69\\6a + 15 = 69\\6a = 69 - 15\\6a = 54\\a = 54 / 6\\a = 9[/tex]

[tex]HIGHEST[/tex] value = [tex]a + 5 = 9 + 5 = 14[/tex]

Hence, [tex]HIGHEST[/tex] value = [tex]14[/tex]

Learn more : https://brainly.com/question/15974141

What is the area of this figure?

Answers

Answer:

90km² only if it is parallelogram

Step-by-step explanation:

base = 9km

height=10km

area of parallelogram = b x h

=9km x 10km

=90km²

Answer:

A = 90km2

Step-by-step explanation:

Area of a rhombus is:

1. A = s x h (if given side and height)

2. A = 1/2 a x b (if given lengths of diagonals)

3. A = s^2 sin A (if given side and length)

Therefore from your problem, height and side is given thus, you'll use number 1

A = s x h

A = 9km x 10km = 90km2

Let V be the set of all 3x3 matrices with Real number entries, with the usual definitions of scalar multiplication and vector addition. Consider whether V is a vector space over C. Mark all true statements (there may be more than one).

a. The scalar closure axiom is satisfied
b. The additive inverse axiom is not satisfied
c The additive inverse axiom is satisfied
d. The additive closure axiom is not satisfied
e. The scalar closure axiom is not satisfied
f. The additive closure axiom is satsified
g. V is not a vector space over C
h. V is a vector space over C
i. The zero axiom is satisfied
j. The zero axiom is not satisfied

Answers

Answer:

the Scalar Closure axiom is not satisfiedV is not a Vector Space of CThe Additive Closure axiom is satisfied.

Step-by-step explanation:

According to the Question,

Given That, Let V be the set of all 3x3 matrices with Real number entries, with the usual definitions of scalar multiplication and vector addition. Consider whether V is a vector space over C.For V is a vector space over C and V is Set of 3x3 Matrices with Real entries.

Then, For any u,w ∈ V ⇒ u+w ∈ V

And u∈V and z∈C ⇒ z u ∈ V

So, If we take any z= i ∈ C

and V be any 3x3 matrices with Real entrices.

then, z,v ∉ V  ∴z,v Has Complex entries

So, the Scalar Closure axiom is not satisfied

Hence, V is not a Vector Space of C

Any u,w ∈ W ⇒ u+w ∈ V

So, The Additive Closure axiom is satisfied.

Please please help me please I really need help please just tap on picture and you will see the question

Answers

Answer:

No , it is not a right angle triangle

Step-by-step explanation:

according to the pythagoras theorem in right angled triangle sum of square of two sides is equal to the square of it's hypotenuse.

using pythagoras theorem

a^2 + b^2 = c^2

9^2 + 16^2 = 25^2

81 + 256 = 625

337 = 625

since sum of square of two smallest sides of a triangle is not equal to the square of it's hypotenuse it can be concluded that the given figure does not form right angle triangle.

Câu 1 (2 điểm). Cho hệ các vector

U = {(1,2,2); (0,-1,-1);(2,3,3);(1,0,0).

a) tìm số chiều và một cơ sở W của không gian con sinh bởi hệ vector U

b) tìm tham số k để u=(2,3,k^2 +1) là một tổ hợp tuyến tính cảu W, và suy ra [u]w

giải hộ mình với

Answers

Answer:

Step-by-step explanation:

Find the area of the following shape:

Answers

Answer:

36cm^2

Step-by-step explanation:

total area: 6x(4+3)=42

total area excluding the space: 42-(2x3)=36

Answer:

36 cm squared

Step-by-step explanation:

To solve this problem, I first construct a line. (shown in yellow in the first photo)

I then find the area of the top rectangle. (6 cm * 4 cm = 24 cm squared.)

Next, I find the area of the lower rectangle. But...to do that I have to find the length of the line that I constructed. To do this, I do  6cm-2cm=4cm.

Then I can find the area of the lower rectangle. (4cm*3cm=12cm squared.)

add up the area of both of the rectangles and.........12+24=36 cm squared

Complete the table of ordered pairs for the linear y=2x-8

Answers

Given:

Consider the below figure attached with this question.

The linear equation is:

[tex]y=2x-8[/tex]

To find:

The values to complete the table of ordered pairs for the given linear equation.

Solution:

We have,

[tex]y=2x-8[/tex]

Substituting [tex]x=0[/tex] in the given equation, we get

[tex]y=2(0)-8[/tex]

[tex]y=0-8[/tex]

[tex]y=-8[/tex]

So, the value for first blank is -8.

Substituting [tex]y=-2[/tex] in the given equation, we get

[tex]-2=2x-8[/tex]

[tex]-2+8=2x[/tex]

[tex]\dfrac{6}{2}=x[/tex]

[tex]3=x[/tex]

So, the value for second blank is 3.

Substituting [tex]x=2[/tex] in the given equation, we get

[tex]y=2(2)-8[/tex]

[tex]y=4-8[/tex]

[tex]y=-4[/tex]

So, the value for third blank is -4.

Therefore, the required complete table is:

     x           y

     0         -8

    3         -2

     2          -4

 Marsha has a bag that contains 4 green marbles, 8 yellow marbles , and 20 red marbles . If she chooses one marble from the bag, what is the probability that the marble is not red?

PLEASE HELP IF YOURE GOOD AT GEOMETRY!!

Answers

Answer:

C. 3/8

HOPE THIS HELPS :)

Answer:

c. 3/8

Step-by-step explanation:

first you need the denomerator by adding all marbles together which equals 32. now for the munerator you need the sum of the green and yellow marbles. this equals 12. so your fraction is 12/32. next we simplify. we can divide both numbers by 4. getting us a fraction of 3/8.

A group of 3 boys is sharing 4 small pans of rice krispie treats. What is the total amount of rice krispie treats each boy will get?

Answers

Answer:

each boy will get 1.33333333333 of a pan

Step-by-step explanation:

Answer:

Each boy in that group would get about 1 and 1 third of a pan of treats or 1.33 repeating

Step-by-step explanation:

To find this you would take the numbers of pans (4) and divide it by the number of boys (3).

4 / 3 = 1.33333

What is the range of f(x) = 3X?
O A. All real numbers greater than 3
O B. All real numbers greater than or equal to 3
O C. All positive real numbers
O D. All real numbers

Answers

D all real numbers (-∞, ∞)

Joe drives for 3 hours and covers 201 miles. In miles per hour, how fast was he driving?​

Answers

Answer:

67 mph

Step-by-step explanation:

201/3 = 67

Write two rational and three irrational number that are between 3and 4

Answers

a)rational number :

1)(3+4 )/2

=7/2 =3.5

2)(3.5+4)/2

=7.5/2 =3.25

b)irrational number :

1)10/3=3.33....

2)11/3=3.66....

......I hope it will help you. ..

Answer:

The real numbers, which can be represented by the ratio of two integer numbers, are called rational numbers, say P/Q where Q is not equal to zero.

The actual numbers that cannot be expressed as the two integer ratio are called irrational numbers.

Step-by-step explanation:

a)rational number :

1)

(3+4 )/2

=7/2 =3.5

2)

(3.5+4)/2

=7.5/2 =3.25

b)irrational number :

1)

10/3=3.33....

2)

11/3=3.66....

3)

√13

Heyy!! Can someone help me please!!

3 (5x + 2) - 2 (4x -4)

I don’t know what to doooo!!

Answers

Answer:

7x + 14

Step-by-step explanation:

the first thing to do is expand the parentheses/brackets.

3(5x + 2) -2(4x - 4) will be

3(5x) + 3(2) -2(4x) -2(-4)

= 15x + 6 - 8x + 8

collect like terms

15x - 8x + 6 + 8 = 7x + 14

the answer is 7x + 14

Answer:

3(5x+2)-2(4x-4)

15x+6-8x+8

15x-8x+6+8

7x+14

Plsssss ans I am suffering

Answers

I. The multiples of three are in the third and sixth column ii. The multiples of 6 are in the sixth column b. The fifth column c. Yes

I need help with this!

Answers

Answer:

1.  y=5   x=1

2.  y=4   x=4/5

3.  y=2   x=2/5

Step-by-step explanation:

Plug the y-values in for y:

1. 5=5x

2. 4=5x

3. 2=5x

Then solve for x:

1. x=1

2. x=4/5

3. x= 2/5

Hope this helps!

Which diagram represents the hypothesis of the converse of corresponding angles theorem?

Answers

Answer:

the first diagram

Step-by-step explanation:

first one

Other Questions
A student constructs a Venn diagram to compare the organelles in plant and animal cells.Venn Diagram of Plant and Animal CellsAnimal OnlyBothPlant OnlyWhich organelle should be listed under "Animal Only" in the diagram? Which of the following is NOT an example of a renewable resource?O biomasswaterairoil 1......me a newsapaper when you're out ?1).will you get 2)Did you get 3)Are you getting 4)were you getting2.Marisa is in her office. She has been there.....7 o'clock1)in2)to3)since4)for3.The school bus.....at 6.001)usually comes2)comes usually 3.)usually come4)come usually Fill in the blanks with the most appropriate words given below.around, react, too, method, fields, on, plants,together, is, family, planter, weeds, experience, talkingEcology is the study of how (1) , animals and people live (2) andhelp each other. But man sometimes forgets he (3) .... a part of the (4).. ofall living things. Here is an example. A planter is (5). about killing (6) on his land. He remembers his first (7). with a weedicide. He was a (8).. then, working (9)....a rubber plantation. He had a problem withweeds. The cost of weeding of the (10). by hand was getting (11)..high. So he used a new, cheap and effective (12), good old SodiumArsenate. He sprayed plenty of this (13) to kill the weeds. The first thing to(14) were the frogs. How do mountains play a role in precipitation patterns?a. As water vapor condenses,the air gets warmer b.it changes the amount of rain received c.it changes the direction of the wind d.as vapor condenses,it depends rain or snow on windward slopes Why is Jeannette more comfortable with Dinita than she is with white children? I wish he was an Englishman who is he 40 ohms1.2 A40 ohms12 VCalculate the total energy developed in 5minutes by the system above. Whats a prepositional phrase 59.73 - 21.876 can u help me How did Greyhound respond to the competition from the new bus lines? Did the intensifiedcompetition ultimately hurt or help Greyhound? Please answer quickly Find the following amount.What is 75% of 16? 5 is subtracted from the sum of 8 and 7. The measures, in degrees, of the three angles of a triangle are given by 2x+1, 3x - 3, and 9x. What is themeasure of the smallest angle? A video store charges $8 per movie, and the fifth movie is free. How much do you actually pay per movie? a) How much electrical energy, in joules, does a 1000W space heater consume when it runs for 8h? (b) How much energy is that in kilowatt hours? (c) Calculate the cost of using this heater if 1kWh costs 0.5 dollars. Use the formula below to find the relative pressure inside the can in psi The name Bleeding Kansas came about as a result of _________________.a.violence between cattle ranchers and railroad buildersb.violence between pro-slavery and anti-slavery supportersc.violence between Native Americans and African Americansd.violence between Kansas and Nebraska residentsPlease select the best answer from the choices provided During the taking of its physical inventory on December 31, 2014, Barry's Bike Shop incorrectly counted its inventory as $229,134 instead of the correct amount of $165,639. The effect on the balance sheet and income statement would be